หัวข้อ: Road to IMO 2017 to Infinity
ดูหนึ่งข้อความ
  #44  
Old 22 ธันวาคม 2016, 18:00
BAWHK BAWHK ไม่อยู่ในระบบ
เริ่มฝึกวรยุทธ์
 
วันที่สมัครสมาชิก: 31 ตุลาคม 2016
ข้อความ: 12
BAWHK is on a distinguished road
Default

ให้ $m,n$ เป็นจำนวนนับโดยที่ $\phi(5^m-1)=5^n-1$ จงแสดงว่า $gcd(m,n)>1$
สมมติ $gcd(m,n)=1$
ให้$p_i$ เป็นจำนวนเฉพาะคี่ที่เเตกต่างกัน เเละ $a_0,a_i$ เป็นจำนวนเต็มบวก สำหรับ $i=1,2,...,k$
ซึ่ง $5^m-1=2^{a_0}p^{a_1}_1...p^{a_k}_k$
จะได้
$5^n-1=\phi(5^m-1)=2^{a_0-1}(p^{a_1}_1-p^{a_1-1}_1)..(p^{a_k}_k-p^{a_k-1}_k)$
เเต่ $gcd(5^m-1,5^n-1)=4$
ทำให้ $a_0=2,a_1=..=a_k=1$ นั่นคือ
$5^m-1=4p_1...p_k$ เเละ $5^n-1=2(p_1-1)...(p_k-1)$
ถ้า m เป็นจำนวนคู่ จะทำให้ $8\mid 5^m-1$ เเต่ $8\nmid 4p_1...p_k$ ขัดเเย้งดังนั้น $m$ เป็นจำนวนคี่
$5^{m+1} \equiv 5 \pmod{p_j}$ ทุก $0<j<k+1$ $\Rightarrow$ $\left(\frac{5}{p_j}\right)=1$ เเละจาก $\left(\frac{5}{p_j}\right) \cdot \left(\frac{p_j}{5}\right) = (-1)^{\frac{5-1}{2}\cdot \frac{p_j-1}{2}}=1$
ดังนั้น $\left(\frac{p_j}{5}\right)=1$ นั่นคือ $p_j \equiv 1,4 \pmod{5} $
ถ้า $p_j \equiv 1 \pmod{5}$ จะทำให้$ 5\mid 5^n-1$ ขัดเเย้ง
ดังนั้น$ p_j \equiv 4 \pmod{5}$ เท่านั้น
ดังนั้น $5^m-1 \equiv 4^{k+1} \pmod{5} \Rightarrow -1 \equiv 4^{k+1} \pmod{5}$ (จะได้ $k$ เป็นจำนวนคู่เท่านั้น) เเละ
$5^n-1 \equiv 2(3^k) \pmod{5} \Rightarrow -1 \equiv 2(3^k) \pmod{5}$
จากทั้ง$2$ จะได้ $3^k \equiv 2(-1)^k \pmod{5} \Rightarrow (-1)^L \equiv 2 \pmod{5}$ เมื่อ $k=2L$ ขัดเเย้ง
ดังนั้น$ gcd(m,n)>1$

22 ธันวาคม 2016 18:01 : ข้อความนี้ถูกแก้ไขแล้ว 1 ครั้ง, ครั้งล่าสุดโดยคุณ BAWHK
ตอบพร้อมอ้างอิงข้อความนี้